Variance of the Euclidean norm under finite moment assumptions











up vote
4
down vote

favorite
2












Let $X = (X_1,X_2 cdots X_n)$ be random vector in $R^n$ with independent coordinate $X_i$ that satisfy $E[X_i^2]=1$ and $E[X_i^4] leq K^4$. Then show that $$operatorname{Var}(| X|_2) leq CK^4$$
where $C$ is a absolute constant and $| |_2$ denotes euclidian norm.



Here is my attempt:
$$begin{align*} E(|X|_2^2 -n)^2 &= E[(sum_{i=1}^n X_i^2)^2 ]-n^2 \
&=E[sum_{i=1}^n X_i^4]+E[sum_{i<j}X_i^2X_j^2] -n^2 \
&leq nK^4 + 2{{n}choose {2}}-n^2 \
&leq n(K^4-1) \
& leq nk^4
end{align*}$$



since $$ E(|X|_2^2 -n)^2 leq nk^4 rightarrow Eleft(frac{|X|_2^2}{n} -1right)^2 leq frac{K^4}{n}$$

and since
$$(forall z geq 0 |z-1|leq |z^2-1|) rightarrow
E(frac{|X|_2}{sqrt n} -1)^2leq E(frac{|X|_2^2}{n} -1)^2 $$



thus:
$$E(frac{|X|_2}{sqrt n} -1)^2 leq K^4/n rightarrow E(|X|_2-sqrt n)^2leq K^4$$



by Jensen inequality:
$$(E[|X|_2] - sqrt n)^2 leq K^4 $$



which is equivalence to
$$ |E[|X|_2] - sqrt n)| leq K^2$$



then when I am trying to bound $Var(| X|_2)$ I meet some problem :



$$operatorname{Var}(| X|_2)=E[|X|_2^2] -(E[|X|_2])^2 leq n- (K^2-sqrt n)^2 leq -K^4+2K^2sqrt n$$ which is not bound by constant , how can I bound that?










share|cite|improve this question
























  • Can you cite the source of your claim? My guess is that $text{Var}|X|$ will grow with $n$.
    – nemo
    Nov 30 at 9:19

















up vote
4
down vote

favorite
2












Let $X = (X_1,X_2 cdots X_n)$ be random vector in $R^n$ with independent coordinate $X_i$ that satisfy $E[X_i^2]=1$ and $E[X_i^4] leq K^4$. Then show that $$operatorname{Var}(| X|_2) leq CK^4$$
where $C$ is a absolute constant and $| |_2$ denotes euclidian norm.



Here is my attempt:
$$begin{align*} E(|X|_2^2 -n)^2 &= E[(sum_{i=1}^n X_i^2)^2 ]-n^2 \
&=E[sum_{i=1}^n X_i^4]+E[sum_{i<j}X_i^2X_j^2] -n^2 \
&leq nK^4 + 2{{n}choose {2}}-n^2 \
&leq n(K^4-1) \
& leq nk^4
end{align*}$$



since $$ E(|X|_2^2 -n)^2 leq nk^4 rightarrow Eleft(frac{|X|_2^2}{n} -1right)^2 leq frac{K^4}{n}$$

and since
$$(forall z geq 0 |z-1|leq |z^2-1|) rightarrow
E(frac{|X|_2}{sqrt n} -1)^2leq E(frac{|X|_2^2}{n} -1)^2 $$



thus:
$$E(frac{|X|_2}{sqrt n} -1)^2 leq K^4/n rightarrow E(|X|_2-sqrt n)^2leq K^4$$



by Jensen inequality:
$$(E[|X|_2] - sqrt n)^2 leq K^4 $$



which is equivalence to
$$ |E[|X|_2] - sqrt n)| leq K^2$$



then when I am trying to bound $Var(| X|_2)$ I meet some problem :



$$operatorname{Var}(| X|_2)=E[|X|_2^2] -(E[|X|_2])^2 leq n- (K^2-sqrt n)^2 leq -K^4+2K^2sqrt n$$ which is not bound by constant , how can I bound that?










share|cite|improve this question
























  • Can you cite the source of your claim? My guess is that $text{Var}|X|$ will grow with $n$.
    – nemo
    Nov 30 at 9:19















up vote
4
down vote

favorite
2









up vote
4
down vote

favorite
2






2





Let $X = (X_1,X_2 cdots X_n)$ be random vector in $R^n$ with independent coordinate $X_i$ that satisfy $E[X_i^2]=1$ and $E[X_i^4] leq K^4$. Then show that $$operatorname{Var}(| X|_2) leq CK^4$$
where $C$ is a absolute constant and $| |_2$ denotes euclidian norm.



Here is my attempt:
$$begin{align*} E(|X|_2^2 -n)^2 &= E[(sum_{i=1}^n X_i^2)^2 ]-n^2 \
&=E[sum_{i=1}^n X_i^4]+E[sum_{i<j}X_i^2X_j^2] -n^2 \
&leq nK^4 + 2{{n}choose {2}}-n^2 \
&leq n(K^4-1) \
& leq nk^4
end{align*}$$



since $$ E(|X|_2^2 -n)^2 leq nk^4 rightarrow Eleft(frac{|X|_2^2}{n} -1right)^2 leq frac{K^4}{n}$$

and since
$$(forall z geq 0 |z-1|leq |z^2-1|) rightarrow
E(frac{|X|_2}{sqrt n} -1)^2leq E(frac{|X|_2^2}{n} -1)^2 $$



thus:
$$E(frac{|X|_2}{sqrt n} -1)^2 leq K^4/n rightarrow E(|X|_2-sqrt n)^2leq K^4$$



by Jensen inequality:
$$(E[|X|_2] - sqrt n)^2 leq K^4 $$



which is equivalence to
$$ |E[|X|_2] - sqrt n)| leq K^2$$



then when I am trying to bound $Var(| X|_2)$ I meet some problem :



$$operatorname{Var}(| X|_2)=E[|X|_2^2] -(E[|X|_2])^2 leq n- (K^2-sqrt n)^2 leq -K^4+2K^2sqrt n$$ which is not bound by constant , how can I bound that?










share|cite|improve this question















Let $X = (X_1,X_2 cdots X_n)$ be random vector in $R^n$ with independent coordinate $X_i$ that satisfy $E[X_i^2]=1$ and $E[X_i^4] leq K^4$. Then show that $$operatorname{Var}(| X|_2) leq CK^4$$
where $C$ is a absolute constant and $| |_2$ denotes euclidian norm.



Here is my attempt:
$$begin{align*} E(|X|_2^2 -n)^2 &= E[(sum_{i=1}^n X_i^2)^2 ]-n^2 \
&=E[sum_{i=1}^n X_i^4]+E[sum_{i<j}X_i^2X_j^2] -n^2 \
&leq nK^4 + 2{{n}choose {2}}-n^2 \
&leq n(K^4-1) \
& leq nk^4
end{align*}$$



since $$ E(|X|_2^2 -n)^2 leq nk^4 rightarrow Eleft(frac{|X|_2^2}{n} -1right)^2 leq frac{K^4}{n}$$

and since
$$(forall z geq 0 |z-1|leq |z^2-1|) rightarrow
E(frac{|X|_2}{sqrt n} -1)^2leq E(frac{|X|_2^2}{n} -1)^2 $$



thus:
$$E(frac{|X|_2}{sqrt n} -1)^2 leq K^4/n rightarrow E(|X|_2-sqrt n)^2leq K^4$$



by Jensen inequality:
$$(E[|X|_2] - sqrt n)^2 leq K^4 $$



which is equivalence to
$$ |E[|X|_2] - sqrt n)| leq K^2$$



then when I am trying to bound $Var(| X|_2)$ I meet some problem :



$$operatorname{Var}(| X|_2)=E[|X|_2^2] -(E[|X|_2])^2 leq n- (K^2-sqrt n)^2 leq -K^4+2K^2sqrt n$$ which is not bound by constant , how can I bound that?







probability random-variables norm variance concentration-of-measure






share|cite|improve this question















share|cite|improve this question













share|cite|improve this question




share|cite|improve this question








edited Nov 26 at 14:29









Davide Giraudo

124k16150258




124k16150258










asked Nov 17 at 21:10









ShaoyuPei

1537




1537












  • Can you cite the source of your claim? My guess is that $text{Var}|X|$ will grow with $n$.
    – nemo
    Nov 30 at 9:19




















  • Can you cite the source of your claim? My guess is that $text{Var}|X|$ will grow with $n$.
    – nemo
    Nov 30 at 9:19


















Can you cite the source of your claim? My guess is that $text{Var}|X|$ will grow with $n$.
– nemo
Nov 30 at 9:19






Can you cite the source of your claim? My guess is that $text{Var}|X|$ will grow with $n$.
– nemo
Nov 30 at 9:19

















active

oldest

votes











Your Answer





StackExchange.ifUsing("editor", function () {
return StackExchange.using("mathjaxEditing", function () {
StackExchange.MarkdownEditor.creationCallbacks.add(function (editor, postfix) {
StackExchange.mathjaxEditing.prepareWmdForMathJax(editor, postfix, [["$", "$"], ["\\(","\\)"]]);
});
});
}, "mathjax-editing");

StackExchange.ready(function() {
var channelOptions = {
tags: "".split(" "),
id: "69"
};
initTagRenderer("".split(" "), "".split(" "), channelOptions);

StackExchange.using("externalEditor", function() {
// Have to fire editor after snippets, if snippets enabled
if (StackExchange.settings.snippets.snippetsEnabled) {
StackExchange.using("snippets", function() {
createEditor();
});
}
else {
createEditor();
}
});

function createEditor() {
StackExchange.prepareEditor({
heartbeatType: 'answer',
convertImagesToLinks: true,
noModals: true,
showLowRepImageUploadWarning: true,
reputationToPostImages: 10,
bindNavPrevention: true,
postfix: "",
imageUploader: {
brandingHtml: "Powered by u003ca class="icon-imgur-white" href="https://imgur.com/"u003eu003c/au003e",
contentPolicyHtml: "User contributions licensed under u003ca href="https://creativecommons.org/licenses/by-sa/3.0/"u003ecc by-sa 3.0 with attribution requiredu003c/au003e u003ca href="https://stackoverflow.com/legal/content-policy"u003e(content policy)u003c/au003e",
allowUrls: true
},
noCode: true, onDemand: true,
discardSelector: ".discard-answer"
,immediatelyShowMarkdownHelp:true
});


}
});














draft saved

draft discarded


















StackExchange.ready(
function () {
StackExchange.openid.initPostLogin('.new-post-login', 'https%3a%2f%2fmath.stackexchange.com%2fquestions%2f3002807%2fvariance-of-the-euclidean-norm-under-finite-moment-assumptions%23new-answer', 'question_page');
}
);

Post as a guest















Required, but never shown






























active

oldest

votes













active

oldest

votes









active

oldest

votes






active

oldest

votes
















draft saved

draft discarded




















































Thanks for contributing an answer to Mathematics Stack Exchange!


  • Please be sure to answer the question. Provide details and share your research!

But avoid



  • Asking for help, clarification, or responding to other answers.

  • Making statements based on opinion; back them up with references or personal experience.


Use MathJax to format equations. MathJax reference.


To learn more, see our tips on writing great answers.





Some of your past answers have not been well-received, and you're in danger of being blocked from answering.


Please pay close attention to the following guidance:


  • Please be sure to answer the question. Provide details and share your research!

But avoid



  • Asking for help, clarification, or responding to other answers.

  • Making statements based on opinion; back them up with references or personal experience.


To learn more, see our tips on writing great answers.




draft saved


draft discarded














StackExchange.ready(
function () {
StackExchange.openid.initPostLogin('.new-post-login', 'https%3a%2f%2fmath.stackexchange.com%2fquestions%2f3002807%2fvariance-of-the-euclidean-norm-under-finite-moment-assumptions%23new-answer', 'question_page');
}
);

Post as a guest















Required, but never shown





















































Required, but never shown














Required, but never shown












Required, but never shown







Required, but never shown

































Required, but never shown














Required, but never shown












Required, but never shown







Required, but never shown







Popular posts from this blog

Plaza Victoria

In PowerPoint, is there a keyboard shortcut for bulleted / numbered list?

How to put 3 figures in Latex with 2 figures side by side and 1 below these side by side images but in...